Skip to content

Commit 7af03ce

Browse files
committed
update for Markov h.w. 1
1 parent 4748dd1 commit 7af03ce

File tree

1,046 files changed

+2262
-1951
lines changed

Some content is hidden

Large Commits have some content hidden by default. Use the searchbox below for content that may be hidden.

1,046 files changed

+2262
-1951
lines changed

.luarc.json

Lines changed: 5 additions & 0 deletions
Original file line numberDiff line numberDiff line change
@@ -0,0 +1,5 @@
1+
{
2+
"Lua.workspace.library": [
3+
"/home/dirichy/Documents/.lib/LuaTeX_Lua-API/library"
4+
]
5+
}

AlgebraicGeometry/1/.number

100644100755
File mode changed.

AlgebraicGeometry/1/solution/.jukit/.jukit_info.json

100644100755
File mode changed.

AlgebraicGeometry/1/solution/AlgebraicGeometry1.pdf

100644100755
12.1 KB
Binary file not shown.

AlgebraicGeometry/1/solution/AlgebraicGeometry1.tex

Lines changed: 4 additions & 5 deletions
Original file line numberDiff line numberDiff line change
@@ -71,10 +71,10 @@
7171
From Lemma \Cref{lem:1} we know $V(I_1\cap I_2)\subset V(I_1I_2)$, so we only need to prove $V(I_1)\cup V(I_2)\subset V(I_1\cap I_2),V(I_1I_2)\subset V(I_1)\cup V(I_2)$.
7272
\begin{itemize}
7373
\item $V(I_1)\cup V(I_2)\subset V(I_1\cap I_2)$:
74-
From \Cref{lem:1} It's obvious.
74+
From \Cref{lem:1} It's obvious.
7575
\item $V(I_1I_2)\subset V(I_1)\cup V(I_2)$:
76-
Consider $p\in V(I_1I_2)$.
77-
If $p\notin V(I_1)\cup V(I_2)$, then $\exists f_1\in I_1,f_2\in I_2, f_1(p)\neq 0,f_2(p)\neq 0$. Now consider $f=f_1f_2\in I_1I_2$, we get $f(p)=f_1(p)f_2(p)\neq 0$, so $p\notin V(I_1I_2)$, it's a contradiction.
76+
Consider $p\in V(I_1I_2)$.
77+
If $p\notin V(I_1)\cup V(I_2)$, then $\exists f_1\in I_1,f_2\in I_2, f_1(p)\neq 0,f_2(p)\neq 0$. Now consider $f=f_1f_2\in I_1I_2$, we get $f(p)=f_1(p)f_2(p)\neq 0$, so $p\notin V(I_1I_2)$, it's a contradiction.
7878
\end{itemize}
7979
\end{solution}
8080

@@ -99,7 +99,6 @@
9999
Then from Lemma\Cref{lem:2} we can get $f=0$. So $I_1I_2=\{0\}$.
100100
Since $p\notin V(I_1),q\notin V(I_2)$, we know $I_1,I_2\neq \{0\}$. So $\exists f_1\in I_1\neq 0,\exists f_2\in I_2\neq 0$, and thus $f=f_1f_2\in I_1I_2\neq 0$, contradiction!
101101
\(\phi\)
102-
So these is not a pair of points can be seperated by two disjoint open set.
102+
So there is not a pair of points can be seperated by two disjoint open set.
103103
\end{solution}
104-
105104
\end{document}

AlgebraicGeometry/10/.number

100644100755
File mode changed.

AlgebraicGeometry/10/solution/AlgebraicGeometry10.pdf

100644100755
File mode changed.

AlgebraicGeometry/10/solution/AlgebraicGeometry10.synctex

100644100755
File mode changed.

AlgebraicGeometry/11/.number

100644100755
File mode changed.

AlgebraicGeometry/11/solution/AlgebraicGeometry11.pdf

100644100755
File mode changed.

AlgebraicGeometry/11/solution/AlgebraicGeometry11.synctex

100644100755
File mode changed.

AlgebraicGeometry/12/.number

100644100755
File mode changed.

AlgebraicGeometry/12/solution/AlgebraicGeometry12.pdf

100644100755
File mode changed.

AlgebraicGeometry/12/solution/AlgebraicGeometry12.synctex

100644100755
File mode changed.

AlgebraicGeometry/13/.number

100644100755
File mode changed.

AlgebraicGeometry/13/solution/AlgebraicGeometry13.pdf

100644100755
File mode changed.

AlgebraicGeometry/2/solution/AlgebraicGeometry2.pdf

100644100755
File mode changed.

AlgebraicGeometry/2/solution/AlgebraicGeometry2.xdv

100644100755
File mode changed.

AlgebraicGeometry/4/.number

100644100755
File mode changed.

AlgebraicGeometry/4/problem/AlgebraicGeometry4.tex

100644100755
File mode changed.

AlgebraicGeometry/4/solution/AlgebraicGeometry4.pdf

100644100755
File mode changed.

AlgebraicGeometry/5/.number

100644100755
File mode changed.

AlgebraicGeometry/5/solution/AlgebraicGeometry5.pdf

100644100755
File mode changed.

AlgebraicGeometry/6/.number

100644100755
File mode changed.

AlgebraicGeometry/6/solution/AlgebraicGeometry6.pdf

100644100755
File mode changed.

AlgebraicGeometry/6/solution/AlgebraicGeometry6.xdv

100644100755
File mode changed.

AlgebraicGeometry/7/.number

100644100755
File mode changed.

AlgebraicGeometry/7/solution/AlgebraicGeometry7.pdf

100644100755
File mode changed.

AlgebraicGeometry/8/.number

100644100755
File mode changed.

AlgebraicGeometry/8/solution/AlgebraicGeometry8.pdf

100644100755
File mode changed.

AlgebraicGeometry/8/solution/AlgebraicGeometry8.synctex

100644100755
File mode changed.

AlgebraicGeometry/9/.number

100644100755
File mode changed.

AlgebraicGeometry/9/solution/AlgebraicGeometry9.pdf

100644100755
File mode changed.

AlgebraicGeometry/9/solution/AlgebraicGeometry9.synctex

100644100755
File mode changed.

AlgebraicGeometry/words.tex

100644100755
File mode changed.

FunctionalAnalysis/6/.number

100644100755
File mode changed.

GroupRepresentation/1/problem/GroupRepresentation1.tex

100644100755
File mode changed.

GroupRepresentation/1/solution/GroupRepresentation1.pdf

100644100755
File mode changed.

GroupRepresentation/1/solution/GroupRepresentation1.synctex

100644100755
File mode changed.

GroupRepresentation/10/.number

100644100755
File mode changed.

GroupRepresentation/10/solution/GroupRepresentation10.pdf

100644100755
File mode changed.

GroupRepresentation/10/solution/GroupRepresentation10.synctex

100644100755
File mode changed.

GroupRepresentation/10/solution/GroupRepresentation10.tex

Lines changed: 5 additions & 5 deletions
Original file line numberDiff line numberDiff line change
@@ -24,7 +24,7 @@
2424
\begin{solution}
2525
Assume \(\{ v_i:i=1,\cdots,n\},\{ w_i:i=1,\cdots ,m\}\) are basis of \(V,W\). Then we get \(\{ v_i \otimes w_j:1 \leq i \leq n,1 \leq j \leq m\}\) is a basis of \(V \otimes W\).
2626
Assume \(\Phi,\Psi,\Gamma\) is the matrix of \(\phi,\psi,\phi \otimes \psi\).
27-
We use \(\{ 1,\cdots,n\}^4\) as the dom of \(\Gamma( g)\).
27+
We use \(\{ 1,\cdots,n\}^4\) as the dom of \(\Gamma( g)\).
2828
Then we get \(( \phi \otimes \psi)( g)( v_i \otimes w_j)=\phi( g)( v_i)\otimes \psi( g)( w_j)\).
2929
So \(\Gamma( g)( e_{ij})=( \sum_{k=1}^{n}\sum_{t=1}^{m} \Phi( g)_{ki} \Psi( g)_{tj} e_{kt})\).
3030
So finally we get \(\Gamma( g)_{k t,i j}=\Phi( g)_{ki}\Psi( g)_{tj}\).
@@ -36,9 +36,9 @@
3636
\end{problem}
3737

3838
\begin{solution}
39-
First since \(x \otimes y = \frac{x}{2} \otimes y + y \otimes \frac{x}{2} + \frac{x}{2}\otimes y - y \otimes \frac{x}{2}\) we get
40-
\(x \otimes y \in \sym^2 V+ \bigwedge^2 V\). Since \(\Span\{ x \otimes y:x,y \in V\}=V \otimes V\), we get
41-
\(V \otimes V = \sym^2 V + \bigwedge^2 V\).
39+
First since \(x \otimes y = \frac{x}{2} \otimes y + y \otimes \frac{x}{2} + \frac{x}{2}\otimes y - y \otimes \frac{x}{2}\) we get
40+
\(x \otimes y \in \sym^2 V+ \bigwedge^2 V\). Since \(\Span\{ x \otimes y:x,y \in V\}=V \otimes V\), we get
41+
\(V \otimes V = \sym^2 V + \bigwedge^2 V\).
4242
Now assume \(\dim V=n\), we only need to prove \(\dim\sym^2 V + \dim \bigwedge^2 V \leq n^2\).
4343
Assume \(\{ v_i:1 \leq i \leq n\}\) is a basis of \(V\), then \(\{v_i \otimes v_j:1 \leq i,j \leq n\}\) is basis of \(V \otimes V\).
4444
Then easily \(\Span\{ v_i \otimes v_j + v_j \otimes v_i:1 \leq i,j \leq n\}=\sym^2 V,\Span\{ v_i \otimes v_j - v_j \otimes v_i : 1 \leq i,j \leq n\}=\bigwedge^2 V\).
@@ -71,7 +71,7 @@
7171
Now we let \(g_1=e,g_2=\sigma,g_3=\sigma^2,g_4=\tau\) and \(W_{ij}=\chi_{i-1}( g_j)\), we have
7272
\[
7373
W=\begin{pmatrix}
74-
1 & 1 & 1 & 1 \\
74+
1 & 1 & 1 & 1 \\
7575
1 & 1 & 1 & -1 \\
7676
2 & 2\cos \frac{2 \pi}{5} & 2 \cos \frac{4 \pi}{5} & 0 \\
7777
2 & 2 \cos \frac{4 \pi}{5} & 2\cos \frac{2 \pi}{5} & 0 \\

GroupRepresentation/11/.number

100644100755
File mode changed.

GroupRepresentation/11/solution/GroupRepresentation11.pdf

100644100755
File mode changed.

GroupRepresentation/11/solution/GroupRepresentation11.synctex

100644100755
File mode changed.

GroupRepresentation/11/solution/GroupRepresentation11.tex

Lines changed: 126 additions & 126 deletions
Large diffs are not rendered by default.

GroupRepresentation/12/.number

100644100755
File mode changed.

GroupRepresentation/12/solution/GroupRepresentation12.pdf

100644100755
File mode changed.

GroupRepresentation/12/solution/GroupRepresentation12.synctex

100644100755
File mode changed.

GroupRepresentation/13/.number

100644100755
File mode changed.

GroupRepresentation/13/solution/GroupRepresentation13.pdf

100644100755
File mode changed.

GroupRepresentation/13/solution/GroupRepresentation13.synctex

100644100755
File mode changed.

GroupRepresentation/2/problem/GroupRepresentation2.tex

100644100755
File mode changed.

GroupRepresentation/2/solution/GroupRepresentation2.pdf

100644100755
File mode changed.

GroupRepresentation/2/solution/GroupRepresentation2.synctex

100644100755
File mode changed.

GroupRepresentation/3/.number

100644100755
File mode changed.

GroupRepresentation/3/problem/GroupRepresentation3.tex

100644100755
File mode changed.

GroupRepresentation/3/solution/GroupRepresentation3.pdf

100644100755
File mode changed.

GroupRepresentation/3/solution/GroupRepresentation3.synctex

100644100755
File mode changed.

GroupRepresentation/4/.number

100644100755
File mode changed.

GroupRepresentation/4/problem/GroupRepresentation4.tex

100644100755
File mode changed.

GroupRepresentation/4/solution/GroupRepresentation4.pdf

100644100755
File mode changed.

GroupRepresentation/4/solution/GroupRepresentation4.synctex

100644100755
File mode changed.

GroupRepresentation/5/.number

100644100755
File mode changed.

GroupRepresentation/5/solution/GroupRepresentation5.pdf

100644100755
File mode changed.

GroupRepresentation/5/solution/GroupRepresentation5.synctex

100644100755
File mode changed.

GroupRepresentation/5/solution/GroupRepresentation5.xdv

100644100755
File mode changed.

GroupRepresentation/6/.number

100644100755
File mode changed.

GroupRepresentation/6/solution/GroupRepresentation6.pdf

100644100755
File mode changed.

GroupRepresentation/6/solution/GroupRepresentation6.synctex

100644100755
File mode changed.

GroupRepresentation/7/.number

100644100755
File mode changed.

GroupRepresentation/7/solution/GroupRepresentation7.pdf

100644100755
File mode changed.

GroupRepresentation/7/solution/GroupRepresentation7.synctex

100644100755
File mode changed.

GroupRepresentation/8/.number

100644100755
File mode changed.

GroupRepresentation/8/solution/GroupRepresentation8.pdf

100644100755
File mode changed.

GroupRepresentation/8/solution/GroupRepresentation8.synctex

100644100755
File mode changed.

GroupRepresentation/9/.number

100644100755
File mode changed.

GroupRepresentation/9/solution/GroupRepresentation9.pdf

100644100755
File mode changed.

GroupRepresentation/9/solution/GroupRepresentation9.synctex

100644100755
File mode changed.

GroupRepresentation/main.tex

100644100755
File mode changed.

MarkovProcess/.subject

Lines changed: 1 addition & 0 deletions
Original file line numberDiff line numberDiff line change
@@ -0,0 +1 @@
1+
MarkovProcess

MarkovProcess/1/.number

Lines changed: 1 addition & 0 deletions
Original file line numberDiff line numberDiff line change
@@ -0,0 +1 @@
1+
1
50.3 KB
Binary file not shown.
Lines changed: 98 additions & 0 deletions
Original file line numberDiff line numberDiff line change
@@ -0,0 +1,98 @@
1+
%!Mode:: "TeX:UTF-8"
2+
%!TEX TS-program = xelatex
3+
\documentclass{ctexart}
4+
\newif\ifpreface
5+
\prefacetrue
6+
\input{../../../global/all}
7+
\begin{document}
8+
\large
9+
\setlength{\baselineskip}{1.2em}
10+
\ifpreface
11+
\input{../../../global/preface}
12+
\else
13+
\maketitle
14+
\fi
15+
\newgeometry{left=2cm,right=2cm,top=2cm,bottom=2cm}
16+
%from_here_to_type
17+
\begin{problem}\label{pro:1}
18+
Assume \((\mathscr{F}_t:t \geq 0,t \in \mathbb{R})\) is a filtration.
19+
For \(t \geq 0\) we let \(\mathscr{F}_{t +}:=\bigcap_{s>t}\mathscr{F}_s\).
20+
Prove that \(\mathscr{F}_t \subset \mathscr{F}_{t +}\) and \((\mathscr{F}_{t +}:t \geq 0)\) is a filtration.
21+
\end{problem}
22+
\begin{solution}
23+
To prove \(\mathscr{F}_t \subset \mathscr{F}_{t +}=\bigcap_{s>t}\mathscr{F}_s\), we only need to prove \(\forall s>t,\mathscr{F}_t \subset \mathscr{F}_s\).
24+
By the definition of filtration it's obvious.
25+
Now we will prove \((\mathscr{F}_{t +}:t \geq 0)\) is a filtration.
26+
Only need to prove \(\forall t,s \in \mathbb{R} \AND t \leq s,\mathscr{F}_{t +}\subset \mathscr{F}_{s +}\).
27+
By the definition of \(\mathscr{F}_{\dot{c} +}\) we know that
28+
\(\mathscr{F}_{t +}=\bigcap_{x>t}\mathscr{F}_x=\bigcap_{x>s}\mathscr{F}_x \cap \bigcap_{x:t<x \leq s}\mathscr{F}_x \subset \bigcap_{x>s}\mathscr{F}_x = \mathscr{F}_{s +}\).
29+
So \((\mathscr{F}_{t +}:t \geq 0)\) is a filtration.
30+
\end{solution}
31+
\begin{problem}\label{pro:2}
32+
Assume \((X_t:t \geq 0,t \in \mathbb{R})\) is a stochastic process on probability space \((\Omega,\mathscr{F},\mathbb{P})\).
33+
Prove that \(\forall s,t \geq 0,\varepsilon >0,\{\rho(X_s,X_t) \geq \varepsilon\} \in \mathscr{F}\).
34+
\end{problem}
35+
\begin{solution}
36+
Easily \(\{\rho(X_s,X_t)\geq \varepsilon\}=\bigcup_{k=1}^{\infty}\{\rho(X_s,X_t)>\varepsilon-\frac{1}{k}\varepsilon\}\).
37+
So we only need to prove \(\forall k \in \mathbb{N}^+,\{\rho(X_s,X_t)>\varepsilon(1-\frac{1}{k})\} \in \mathscr{F}\).
38+
Take \(\delta=\varepsilon(1-\frac{1}{k})\), only need to prove \(\forall \delta>0,\{\rho(X_s,X_t)>\delta\}\in \mathscr{F}\).
39+
40+
\(\forall t \geq 0,X_t:\Omega \to E\) is measurable, where \(E \subset \mathbb{R}^d\).
41+
So we can find a countable dense set in \(\mathbb{R}^d\), write \(D\).
42+
We will prove that \(\{\rho(X_s,X_t)>\delta\}=\bigcup_{q \in D} \{\rho(X_s,q)-\rho(X_t,q)>\delta\}\).
43+
On one hand, easily \(\rho(X_s,q)-\rho(X_t,q)>\delta \implies \rho(X_s,X_t)>\delta\) from triangle inequality.
44+
So we easily get \(\{\rho(X_s,X_t)>\delta\}\supset\bigcup_{q \in D} \{\rho(X_s,q)-\rho(X_t,q)>\delta\}\).
45+
On the other hand, assume for certain \(\omega \in \Omega\) we have \(\rho(X_s(\omega),X_t(\omega))>\delta\), we will prove \(\exists q \in D,\rho(X_s(\omega),q)-\rho(X_t(\omega),q)>\delta\).
46+
For convenience, we omit \((\omega)\) from now on to the end of this paragraph.
47+
Since \(\rho(X_s,X_t)>\delta\), we know \(\gamma:=\frac{\rho(X_s,X_t)-\delta}{2}>0\).
48+
Since \(D\) is dense, we obtain \(\exists q \in D,\rho(X_t,q)<\gamma\).
49+
So from triangle inequality we get \(\rho(X_s,q)\geq\rho(X_s,X_t)-\rho(X_t,q)>2 \gamma+\delta-\gamma=\gamma+\delta\).
50+
So we get \(\rho(X_s,q)-\rho(X_t,q)>\gamma + \delta - \gamma =\delta\).
51+
Finally, we get \(\{\rho(X_s,X_t)>\delta\}= \bigcup_{q \in D}\{\rho(X_s,q)-\rho(X_t,q)>\delta\}\).
52+
53+
Noting \(\bigcup_{q \in D}\{\rho(X_s,q)-\rho(X_t,q)>\delta\}=\bigcup_{q \in D}\bigcup_{p \in \mathbb{Q}^+}\{\rho(X_s,q)>\delta+p,\rho(X_t,q)<p\}\),
54+
and \(D,\mathbb{Q}^+\) are countable, so we only need to check \(\{\rho(X_s,q)>\delta+p,\rho(X_t,q)<p\} \in \mathscr{F},\forall q \in D,p \in \mathbb{Q}^+\).
55+
Noting \(\{\rho(X_s,q)>\delta+p,\rho(X_t,q)<p\}=\{\rho(X_s,q)>\delta+p\}\cap\{\rho(X_t,q)<p\}\),
56+
and \(X_s,X_t\) are measurable from \(\Omega\) to \(E\), we obtain \(\{\rho(X_s,q)>\delta+p\},\{\rho(X_t,q)<p\} \in \mathscr{F}\).
57+
So we proved \(\{\rho(X_s,X_t) >\delta\} \in \mathscr{F},\forall s,t \geq 0,\forall \delta>0\).
58+
59+
Finally, we obtain \(\{\rho(X_s,X_t)\geq \varepsilon\} \in \mathscr{F},\forall s,t \geq 0,\varepsilon>0\).
60+
\end{solution}
61+
\begin{problem}\label{pro:3}
62+
Let \(\mathscr{D}_X:=\{\mu_J^X:J \in S(I)\}\) be the family of finite-dimentional distributions of a stochastic process \((X_t:t \geq 0,t \in \mathbb{R})\).
63+
\(\forall (s_1,s_2)\in S(I)\) and \(J=(t_1,\cdots,t_n)\in S(I)\), write \(K_1:=(s_1,s_2,t_1,\cdots,t_n) \in S(I),K_2:=(s_2,s_1,t_1,\cdots,t_n) \in S(I)\).
64+
Take \(A_1,A_2 \in \mathscr{E},B \in \mathscr{E}^n\), prove that
65+
\[
66+
\mu^X_{K_1}(A_1 \times A_2 \times B)=\mu^X_{K_2}(A_2 \times A_1 \times B)
67+
\]
68+
and
69+
\[
70+
\mu^X_{K_1}(E \times E \times B)=\mu^X_{K_2}(E \times E \times B)=\mu^X_{J}(B)
71+
\]
72+
\end{problem}
73+
74+
\begin{problem}\label{pro:4}
75+
Assume \((\tau_k:k \in \mathbb{N}^+)\) is an i.i.d sequence of r.v. with exponential distribution with parameter \(\alpha>0\).
76+
Let \(S_n:=\sum_{k=1}^{n}\tau_k\). For \(t \geq 0,t \in \mathbb{R}\), let:
77+
\[
78+
N_t:=\sum_{n=1}^{\infty}\mathbbm{1}_{\{S_n \leq t\}},
79+
X_t:=\sum_{n=1}^{\infty}\mathbbm{1}_{\{S_n < t\}}
80+
\]
81+
Prove that \(N\) and \(X\) are modifications of each other, but they are not indistinguishable.
82+
\end{problem}
83+
84+
\begin{problem}\label{pro:5}
85+
Assume \(T\) is non-negetive r.v. with distribution function \(F\) continuous on \(\mathbb{R}\).
86+
Let \(X_t=\mathbbm{1}_{\{T \leq t\}}\).
87+
Prove that \(X\) is stochastically continuous.
88+
\end{problem}
89+
\begin{problem}\label{pro:6}
90+
Assume \(I=\mathbb{Z}^+\), then the stochastic process \(X=(X_0,X_1,\cdots)\) is a r.v. from \(\Omega\) to \(E^\infty\).
91+
Define the distribution of \(X\), \(\mu_X\), as follows:
92+
\[
93+
\mu_X(A)=\mathbb{P}(X \in A),A \in \mathscr{E}^\infty
94+
\]
95+
Then stochastic process \(X,Y\) are indistinguishable \(\iff \mu_X=\mu_Y\).
96+
\end{problem}
97+
98+
\end{document}

MarkovProcess/localcommands

Whitespace-only changes.

MarkovProcess/template

Lines changed: 18 additions & 0 deletions
Original file line numberDiff line numberDiff line change
@@ -0,0 +1,18 @@
1+
%!Mode:: "TeX:UTF-8"
2+
%!TEX TS-program = xelatex
3+
\documentclass{ctexart}
4+
\newif\ifpreface
5+
\prefacetrue
6+
\input{../../../global/all}
7+
\begin{document}
8+
\large
9+
\setlength{\baselineskip}{1.2em}
10+
\ifpreface
11+
\input{../../../global/preface}
12+
\else
13+
\maketitle
14+
\fi
15+
\newgeometry{left=2cm,right=2cm,top=2cm,bottom=2cm}
16+
%from_here_to_type
17+
18+
\end{document}

ParallelComputing.zip

9.15 MB
Binary file not shown.

ParallelComputing/BRW_revised.zip

-2.36 KB
Binary file not shown.

ParallelComputing/ReportTemplate.pdf

2.35 MB
Binary file not shown.

0 commit comments

Comments
 (0)